Tải bản đầy đủ (.doc) (12 trang)

suu tam duoc mot so de thi HSG hay

Bạn đang xem bản rút gọn của tài liệu. Xem và tải ngay bản đầy đủ của tài liệu tại đây (275.25 KB, 12 trang )

Bài 1(2) : Tìm tất cả các số chính phương dạng .
Lời giải :
Do là số chính phương nên = k
2
.
Lại vì : 10001 ≤ ≤ 99999, => 101 ≤ k < 317. Vậy k phải là số có ba chữ số. Đặt k = . Cũng
từ giả thiết bài toán là số chính phương nên a thuộc {1, 4, 5, 6, 9}.
+) Nếu a = 1 thì : 100 < < 142 (trong đó p = 1 hoặc p = 9)
Với p = 1, ta có các số thỏa mãn điều kiện bài toán là : 101
2
= 10201, 111
2
= 12321, 121
2
= 14641 ;
còn với p = 9 ta thấy không có số nào thỏa mãn điều kiện đề bài.
+) Nếu a = 4 thì : 200 < < 224 (p = 2 hoặc p = 8). Thử trực tiếp ta có các số thỏa mãn là :
202
2
= 40804, 212
2
= 44944.
+) Nếu a = 5 thì : 223 < < 245 (trong đó p = 5). Trong trường hợp này không có số nào thỏa
mãn điều kiện đề bài.
+) Nếu a = 6 thì : 244 < < 265 (p = 4 hoặc p = 6). Chỉ có một số thỏa mãn trường hợp này là :
264
2
= 69696.
+) Nếu a = 9 thì : 300 < < 317 (p = 3 hoặc p = 7). Trường hợp này cũng chỉ có một số thỏa mãn
là : 307
2


= 94249. Tóm lại có 7 số thỏa mãn điều kiện bài ra : 10201, 12321, 40804, 14641, 44844,
69696, 94249.
Nhận xét :
- Nếu bài toán có thêm điều kiện a, b, c đôi một khác nhau thì chỉ có 5 số thỏa mãn đề bài.
Bài 3(2) : Cho tam giác ABC. Các điểm M, N theo thứ tự thuộc các cạnh AB, AC sao cho diện tích
tam giác AMN bằng một nửa diện tích tam giác ABC (M ≠ B ; N ≠ C). Chứng minh : Trọng tâm của
tam giác ABC nằm trong tam giác AMN.
Lời giải : (của bạn Nguyễn Thị Quỳnh Mai, 6B, THCS Bán công Hương Khê, Hà Tĩnh)
Gọi G là trọng tâm ABC. Đặt L là giao điểm của BG và AC ; O là giao điểm của BL và MN.
Ta có : AL = CL ; GB/GL = 2 (1)
Theo giả thiết : S
(AMN)
= 1/2 . S
(ABC)
Mặt khác, vì AL = CL nên : S
(ABL)
= 1/2 . S
(ABC)

Vậy S
(AMN)
= S
(ABL)
=> S
(OLN)
= S
(OMB)

=> S
(BLN)

= S
(NMB)
=> ML // BN
=> : OB/OL = BN/ML = AN/AL < AC/AL = 2 (2) (định lí Talét)
Từ (1), (2) => :
OB/OL < GB/GL => OB/OL + 1 < GB/GL + 1 => BL/OL < BL/GL
=> GL < OL => G thuộc đoạn OL => G thuộc tam giác AMN (đpcm).
Nhận xét :
1) Bài này có khá nhiều bạn tham gia giải, tất cả đều giải đúng. Tuy nhiên, nhiều bạn giải quá dài và
phức tạp.
2) Ngoài cách giải trên còn một hướng giải khác nữa.
Đặt P là giao điểm của AG và BC ; Q là giao điểm của AP và MN.
Ta có đẳng thức quen thuộc :
AB/AM + AC/AN = 2.AP/AQ (3)
Theo giả thiết :
S
(ABC)
/ S
(AMN)
= 2 => AB/AM . AC/AN = 2 (4)
Từ (3), (4), sau một vài biến đổi đại số, ta có : AG < AQ. Từ đó => đpcm.
Bài 4(2) : Giải phương trình :
x
2
+ 2x + 3 = (x
2
+ x + 1) (x
4
+ x
2

+ 4)
Lời giải : (của bạn Phan Việt Thành, 8D, THCS Lương Thế Vinh, Tuy Hòa, Phú Yên)
Ta có : x
2
+ x + 1 = (x + 1/2)
2
+ 3/4 > 0 với mọi x
Mặt khác : Với mọi x ta có x
4
+ x
2
+ 4 ≥ 4
=> (x
2
+ x + 1) (x
4
+ x
2
+ 4) ≥ 4 (x
2
+ x + 1)
= x
2
+ 2x + 3 + 2x
2
+ (x + 1)
2
> x
2
+ 2x + 3

Chứng tỏ phương trình vô nghiệm.
Bài 5(2) : Tìm x, y để biểu thức :
đạt giá trị nhỏ nhất.
Lời giải :
Ta có:
Dấu “=” xảy ra khi và chỉ khi y = -1 và 3 ≥ x ≥ -1.
Bài 1 (3) : Tìm tất cả các cặp số nguyên không âm (x, y) sao cho : x - y = x
2
+ xy + y
2
.
Lời giải :
Giả sử có cặp số nguyên không âm (x, y) thỏa mãn hệ thức x - y = x
2
+ xy + y
2
(1)
Từ đó dễ dàng => rằng x ≥ x - y = x
2
+ xy + y
2
≥ 3xy (2)
- Nếu x = 0, từ (1) ta có -y = y
2
=> y = 0.
- Nếu x ≠ 0, thay x = 1 vào (2) => 1 ≥ 3y => y = 0, thay y = 0 vào (1) => x = x
2
=> x = 1.
Tóm lại : Có hai cặp số (0, 0); (1, 0) thỏa mãn đề bài.
Bài 2(4) : Cho a, b, c là 3 số thỏa mãn điều kiện :

Tính tổng : a
2001
+ b
2002
+ c
2003
.
Lời giải :
Từ (1) => a, b, c ≤ 1. Trừ từng vế của (1) cho (2) ta có : a
2002
(1 - a) + b
2002
(1 - b) + c
2002
(1 - c) = 0 (3)
Vì a, b, c ≤ 1 nên a
2002
(1 - a) ≥ 0 ; b
2002
(1 - b) ≥ 0 ; c
2002
(1 - c) ≥ 0.
Từ đó ta có a
2001
= a
2003
; b
2002
= b
2003

.
=> : a
2001
+ b
2002
+ c
2003
= a
2003
+ b
2003
+ c
2003
= 1
Nhận xét :
1/ Từ lời giải trên, kết hợp với (1) hoặc (2) có thể => trong ba số a, b, c có đúng một số bằng 1 và
hai số còn lại bằng 0.
2/ Một số bạn từ (1) => 0 ≤ a ≤ 1 là sai. Một số bạn tự thêm giả thiết a, b, c là các số nguyên dương.
Bài 4(4) : Cho ΔABC nhọn, ba đường cao AD, BE và CF cắt nhau tại H. Qua A vẽ các đường thẳng
song song với BE, CF lần lượt cắt các đường thẳng CF, BE tại P và Q. Chứng minh rằng PQ vuông
góc với trung tuyến AM của ΔABC.
Lời giải :
Vì ΔABC nhọn nên trực tâm H nằm trong ΔABC (hình dưới).
Lần lượt đặt I là giao điểm của AH và PQ ; K là giao điểm của AM và PQ. Từ giả thiết => tứ giác
APHQ là hình bình hành, nên I là trung điểm của AH.
Cũng từ giả thiết => AP vuông góc với AC ; AQ vuông góc với AB => ∠ BAC =∠ AQH; ∠ ACB
= ∠ AHQ (các cặp góc có hai cạnh tương ứng vuông góc)
=> ΔABC đồng dạng với ΔQAH (g.g) => AB/QA = BC/AH = AC/AH .
Mặt khác M, I lần lượt là trung điểm của BC, AH nên :
AM/QI = BC/AH = MC/IH => AM/QI = MC/IH = AC/QH

=> ΔAMC đồng dạng với ΔQIH (c.c.c)
=> ∠ CAM = ∠ HQI hay ∠ EAK = ∠ EQK => tứ giác KAQE nội tiếp
=> ∠ AKQ = ∠ AEQ = 90
o
(cùng chắn cung AQ, BQ vuông góc với AC) => PQ vuông góc với
AM (đpcm).
Bài 3(5) : So sánh A và B biết :
A = (2003
2002
+ 2002
2002
)
2003

B = (2003
2003
+ 2002
2003
)
2002

Lời giải : (của bạn Võ Văn Tuấn)
Ta sẽ chứng minh bài toán tổng quát :
(a
n
+ b
n
)
n + 1
> (a

n + 1
+ b
n + 1
)
n
với a, b, n là các số nguyên dương.
Thật vậy, không mất tính tổng quát, giả sử a ≥ b. (a
n
+ b
n
)
n + 1
= (a
n
+ b
n
)
n
.(a
n
+ b
n
) > (a
n
+ b
n
)
n
.a
n

=
[(a
n
+ b
n
)a]
n
= (a
n
.a + b
n
.a)
n
≥ (a
n
.a + b
n
.b)
n
= (a
n + 1
+ b
n + 1
)
n
.
Với a = 2003, b = n = 2002, ta có A > B.
Bài 4(5) : Tam giác ABC có E là trung điểm cạnh BC sao cho ∠ EAB = 15
o
, ∠ EAC = 30 . Tính ∠

C.
Lời giải : (của bạn Phạm Thị Hồng Anh)
Gọi F là điểm đối xứng của C qua AE và I là giao điểm của CF và AE, => AI vuông góc với CI.
Xét tam giác vuông IAC, vuông tại I, có ∠ IAC = 30
o
=> ∠ ACF = ∠ ACI = 60
o
(1) .
Ta có AI là trung trực của CF nên ΔAFC cân, từ (1) => ΔAFC đều.
Nhận xét rằng, IE là đường trung bình của ΔBFC nên IE // FB, mà IE vuông góc với FC => BF
vuông góc với FC hay ΔBFC vuông tại F => góc BFC = 90
o
; ΔAFC đều => ∠ CFA =∠ CAF =
60
o
, => :
∠ BFA = ∠ BFC + CFA = 90
o
+ 60
o
= 150
o

∠ FAB = ∠ CAF - ∠ CFA - ∠ BAE = 60
o
- 30
o
- 15
o
= 15

o
.
Xét ΔFBA, ∠ BFA = 150
o
và ∠ FAB = 15
o
. => ∠ FBA = 15
o
su ra ΔFBA cân tại F => FB = FA =
FC. Từ đó, ΔBFC vuông cân tại F => ∠ BCF = 45
o
(2)
Từ (1), (2) => ∠ ACB = ∠ ACF + ∠ BCF = 60
o
+ 45
o
= 105
o
.
Vậy ∠ C = 105
o
.
Bài 2(7) : Cho a, b, c thỏa mãn : a/2002 = b/2003 = c/2004 .
Chứng minh rằng : 4(a - b)(b - c) = (c - a)
2

Lời giải : Theo tính chất của tỉ lệ thức ta có :
a/2002 = b/2003 = c/2004 = (a - b)/(2002 - 2003) = (b - c)/(2003 - 2004) = (c - a)/(2004 - 2002)
=> : (a - b)/-1 = (b - c)/-1 = (c - a)/2
=> : (a - b)(b - c) = [ (c - a)/2 ]

2
=> 4(a - b)(b - c) = (c - a)
2
(đpcm).
Bài 4(8) : Cho hình chữ nhật ABCD (như hình vẽ), biết rằng AB = 30 cm, AD = 20 cm, AM = 10
cm, BP = 5 cm, AQ = 15 cm. Tính diện tích tam giác MRS.
Lời giải : Giả sử đường thẳng PQ cắt các đường thẳng CD, AB tương ứng tại E, F.
Ta có : DE/EC = QD/CP = 5/15 = 1/3 , hay DE/DC = 1/2 => DE = 15 cm.
Ta thấy tam giác QDE = tam giác PBF => BF = DE = 15 cm.
Lại có : DE/MF = DR/MR = 15/35 = 3/7 => MR/MD = 7/10 (1).
Vì MS/MC = MF/EC = 35/45 = 7/9 => MS/MC = 7/16 (2)
Ta có : S
MDC
= S
ABCD
- SAMD - S
BMC
= AB.AD - 1/2AD.(AM + MB) = 300 (cm
2
) (3)
Mặt khác, từ (1) và (2) ta có :
S
MRS
/S
MCD
= (MR/MD).(MS/MC) = 7/10 . 7/16 = 47/160
Do đó, từ (3) ta có :
S
MRS
= 300. (49/160) = 91,875 (cm

2
).
Bài 5(8) : Cho tam giác ABC không vuông. Các đường cao BB’, CC’ cắt nhau tại H. Gọi K là trung
điểm của AH, I là giao điểm của AH và B’C’. Chứng minh rằng : I là trực tâm của tam giác KBC.
Lời giải :
Trường hợp 1 : Tam giác ABC nhọn (hình vẽ). Gọi L là điểm đối xứng của H qua BC.
Ta có : ∠ BLC = ∠ BHC = ∠ B ' HC ' (đối đỉnh) = 180
o
- ∠ BAC (Vì AC 'HB ' nội tiếp)
=> ∠ BLC + ∠ BAC = 180
o

=> tứ giác ABLC nội tiếp
=> ∠ CLA = ∠ CBA . (1)
Mặt khác, vì ∠ BB 'C = ∠ BC 'C ( = 90
o
) nên tứ giác BC’B’C nội tiếp => ∠ AB ' C ' = ∠ CBC ' .
(2)
Từ (1) và (2) => : ∠ CLI = ∠ AB 'I => tứ giác CLIB ’ nội tiếp => ∠ B 'CI = ∠ B 'LI . (3)
Theo giả thiết, tam giác AHB’ vuông tại B’, B’K là trung tuyến nên KB’ = KH => ∠ KB 'H = ∠
KHB ' = ∠ BHL = ∠ BLH (vì tam giác BLH cân tại B) => tứ giác KB’LB nội tiếp , suy ra ∠ B 'BK
= ∠ B 'LK . (4)
Từ (3) và (4) => tứ giác BCB’E nội tiếp (E là giao của CI và BK)
=> ∠ BEC = ∠ BB 'C = 90
o
=> CI vuông góc với BK.
Chú ý rằng : KI vuông góc với BC
Vậy I là trực tâm của tam giác KBC.
Trường hợp 2 : Tam giác ABC tù, chứng minh hoàn toàn tương tự như trường hợp 1.
Bài 1(11) : Phân tích số 8030028 thành tổng của 2004 số tự nhiên chẵn liên tiếp.

Lời giải :
Ta thấy : Tổng của 2004 số tự nhiên chẵn liên tiếp là S = a + (a + 2) + + (a + 4006) = [ a + (a +
4006)] : 2 x 2004 = (a + 2003) x 2004.
Do đó S = 8030028 tương đương với (a + 2003) x 2004 = 8030028 hay a = 2004.
Vậy 8030028 = 2004 + 2006 + 2008 + + 6010.
Bài 2(11) : Tìm số nguyên a lớn nhất sao cho số T = 4
27
+ 4
1016
+ 4
a
là số chính phương.
Lời giải : Ta xét a là số nguyên thỏa mãn a ≥ 27 và T là số chính phương. Nhận xét T = 4
27
(1 + 4
989
+ 4
a - 27
) = (2
27
)
2
. (1 + 2
1978
+ (2
a - 27
)
2
), => S = 1 + 2
1978

+ (2
a - 27
)
2
là số chính phương.
Chú ý : 1 + 2
1978
+ (2
a - 27
)
2
> (2
a - 27
)
2
=> 1 + 2
1978
+ (2
a - 27
)
2
≥ (2
a - 27
+ 1)
2

Tức là ta có 2
1978
≥ 2.2
a - 27


=> 1978 ≥ a - 26 => 2004 ≥ a.
Với a = 2004 thì T = (2
27
)
2
. (2
1977
+ 1)
2
là số chính phương.
Vậy số nguyên a lớn nhất cần tìm là a = 2004
Bài 4(11) : Tính góc A của tam giác ABC biết rằng ∠ O
1
OO
2
= 90
o
với O
1
, O, O
2
lần lượt là tâm
của các đường tròn nội tiếp, ngoại tiếp và bàng tiếp (trong góc A) của tam giác ABC.
Lời giải :Gọi I là giao điểm của AO
2
với đường tròn (O).
Ta thấy : ∠ IBO
1
= ∠ IO

1
B ( = (∠ A + ∠ B)/2 ) => ΔIBO
1
cân tại I, từ đó IB = IO
1
(1).
Mặt khác ∠ O
1
BO
2
= 90
o
nên ∠ IBO
2
= ∠ IO
2
B hay ΔIBO
2
cân tại I => IB = IO
2
(2).
Từ (1), (2) => IO
1
= IO
2
.
Theo giả thiết ∠ O
1
OO
2

= 90
o
=> OI = 1/2.O
1
O
2
.
Do đó OI = BI = OB = R (bán kính của đường tròn (O)) => ΔBIO đều => ∠ BOI = 60
o
, => ∠ BAI
= 30
o
. Vậy ∠ BAC = 60
o
.
Bài 5(11) : Về phía ngoài của tam giác ABC ta dựng các tam giác vuông đồng dạng ABE, ACF (∠
ABE = ∠ ACF = 90
o
).
Chứng minh rằng : BF, CE và đường cao AH của tam giác đồng quy.
Lời giải :
Cách 1 : (của bạn Võ Văn Tuấn, 7A5, THCS Buôn Hồ, KRông Buk, Đắk Lắk)
Vì ΔBEA đồng dạng với ΔCFA nên AB / EB = AC / FC.
Trên tia đối của tia AH lấy điểm K sao cho : AK = AB/EB.BC = AC/FC.CB (hình 1)
Vì AK = AB/EB.BC nên AK/AB = BC/BE (1)
Mặt khác : ∠ KAB = ∠ CBE (2) (hai góc có cạnh tương ứng vuông góc và cùng tù)
Từ (1), (2) => : ΔKAB đồng dạng với ΔCBE
=> :∠ ABK = ∠ BEC => ∠ ABK + ∠ EBK = ∠ BEC + ∠ EBK
=> : 90
o

= ∠ ENB (N là giao điểm của EC và BK) => CE vuông góc với BK.
Tương tự như vậy : BF vuông góc với CK.
Vậy BF, CE, AH là ba đường cao của ΔBCK => BF, CE, AH đồng quy.
Cách 2 : (của các bạn Huỳnh Quốc Uy và Trần Lương Khiêm, 9A, THCS Trần Hưng Đạo, Quảng
Ngãi)
Đặt I = BF giao với CE. Gọi M, N là hình chiếu của A trên CE, BF (hình 2).
Dễ thấy các tứ giác AMBE, ANCF, AMIN, AMHC nội tiếp. Vì các tứ giác AMBE, ANCF nội tiếp
và vì ΔABE đồng dạng với ΔACF nên ta có :
∠ BME = ∠ BAE = ∠ CAF = ∠ CNF => ∠ BMC = ∠ BNC
=> Tứ giác BMNC nội tiếp => ∠ MNB = ∠ MCB (1)
Vì tứ giác AMIN nội tiếp nên : ∠ MNB = ∠ MAI (2)
Vì tứ giác AMHC nội tiếp nên : ∠ MCB = ∠ MAH (3)
Từ (1), (2), (3) => : ∠ MAI = ∠ MAH => tia AI trùng tia AH => I thuộc AH => BF, CE, AH đồng
quy.
Trên đây, ta mới chỉ vẽ hình và giải bài toán trong trường hợp các góc ABC, ACB < 90
o
. Nếu ∠
ACB ≥ 90
o
hoặc ∠ ACB ≥ 90
o
thì ta cũng có lời giải tương tự.
Bài 1(12) : Cho số tự nhiên N = 2003
2004
. Viết N thành tổng của k số tự nhiên nào đó n
1
, n
2
, …, n
k

. S
= n
1
3
+ n
2
3
+ … + n
k
3
. Tìm số dư của phép chia S cho 6.
Lời giải :
Vì a
3
- a = a.(a
2
-1) = (a - 1).a.(a + 1) là tích của ba số nguyên liên tiếp nên a
3
- a chia hết cho 6 với
mọi số nguyên a.
Đặt N = n
1
+ n
2
+ … + n
k
, ta có :
S - N = (n
1
3

+ n
2
3
+ … + n
k
3
) - (n
1
+ n
2
+ … + n
k
) = (n
1
3
- n
1
) + (n
1
3
- n
1
) + … + (n
k
3
- n
k
) chia hết cho
6 => S và N có cùng số dư khi chia cho 6.
Mặt khác, 2003 chia cho 6 dư 5 => 2003

2
chia cho 6 dư 1 => N = 2003
2004
= (2003
2
)
1002
chia cho 6
dư 1. Vậy : S chia cho 6 dư 1.
Bài 4(12) : Cho hình thang vuông ABCD có AD // BC, AB vuông góc với AD, AD = 4 cm, AB =
BC = 2 cm. Hãy tìm một con đường ngắn nhất đi từ đỉnh A tới một điểm M trên cạnh DC, rồi tới
điểm N trên cạnh AB, quay lại một điểm P trên cạnh DC và trở về A.
Lời giải : Bài toán đưa về tìm giá trị nhỏ của tổng T = AM + MN + NP + PA.
Ta cần kết quả sau.
Bổ đề : Trong một hình thang vuông, độ dài đoạn thẳng nối hai điểm nằm trên hai cạnh bên không
nhỏ thua độ dài đáy nhỏ (các bạn tự chứng minh bổ đề này).
áp dụng bổ đề, ta có : MN ≥ BC ; NP ≥ BC => MN + NP ≥ 2BC (1).
Dựng CH cuông góc với AD. Nhận thấy tứ giác ABCH là hình vuông, => : CH = AH = AB = BC =
2 cm => DH = 2 cm.
Ta có CH = AH = HD = 2 cm => ΔCAD vuông cân tại C.
Vì P, M thuộc CD => PA ≥ AC ; AM ≥ AC => AM + PA ≥ 2AC (2).
Từ (1), (2) => T ≥ 2(AC + BC) =
đẳng thức xảy ra khi và chỉ khi P, M, C trùng nhau và N trùng với B.
Vậy con đường ngắn nhất thỏa mãn điều kiện đề bài dài chính là con đường : A đến C
đến B đến C đến A.
Bài 5(13) : Cho hình thang ABCD có AB song song và bằng một nửa CD. H là trung điểm của CD.
Điểm M nằm ngoài hình thang sao cho MH vuông góc và bằng một phần tư CD.
Bên ngoài hình thang, ta dựng các tam giác ADE, BCF vuông cân tại E, F. Chứng minh rằng tam
giác MEF vuông cân tại M.
Lời giải :

Cách 1 : (của bạn Chu Toàn Thắng) Trước hết, ta nhắc lại một bổ đề quen thuộc.
Bổ đề (*) : Cho tam giác ABC. Về phía ngoài của tam giác, ta dựng các tam giác ABE, ACF vuông
cân tại E, F. M là trung điểm của AB. Khi đó, tam giác MEF vuông cân.
Trở lại bài toán.
Trên tia đối của tia MD, lấy điểm K sao cho : MK = MD (hình 1). Dễ thấy : KC vuông góc với DC
(1) và CK = 2MH (2)
dd>Từ (1) => : ∠ KCF = 360
o
- ∠ KCD - ∠ DCB - ∠ BCF = 360
o
- 90
o
-(180
o
- ∠ CBA) - 45
o
= 45
o
+ ∠ CBA = ∠ FBC + ∠ CBA = ∠ ABF Vậy: ∠ KCF = ∠ ABF
Từ (2) => : KC = 1/2 CD = AB (4)
Từ (3), (4) và CF = BF (do tam giácBCF vuông cân tại F) ta có : Δ KCF = Δ ABF (c.g.c)
=> FK = FA ; ∠ CFK = ∠ AFB
=> ∠ KFA = ∠ CFB + ∠ CFK - ∠ BFA = ∠ CFB = 90
0

=> Δ AKF vuông cân tại F.
áp dụng bổ đề (*) cho tam giác ADK và các tam giác ADE, AKF vuông cân tại E, F ta có : Δ MEF
vuông cân tại M.
Cách 2 : (của bạn Nguyễn Tài Đại)
Gọi K là trung điểm của AB. Lấy điểm N nằm ngoài hình thang ABCD sao cho NK = AB ; NK

vuông góc với AB (hình 2).
Dễ thấy : tam giác MHC = tam giác BKN => MC = NB ; mặt khác, AB // CD và tam giác BCF
vuông cân tại F nên Δ FMC = Δ FNB (c.g.c) => Δ MFN vuông cân tại F (bạn đọc tự chứng minh).
Tương tự như vậy, Δ MEN vuông cân tại E.
=> MENF là hình vuông => tam giác MEF vuông cân tại M.
Bài 1(14) : Cho x ; y thỏa mãn :
Hãy tính giá trị của biểu thức sau :
T = x
2003
+ y
2003
.
Lời giải : Nhận xét :
Do đó kết hợp với giả thiết ta suy ra :
Cộng theo từng vế của hai đẳng thức trên ta có : y + x = - x - y.
Suy ra x + y = 0. Do đó T = x
2003
+ y
2003
= x
2003
+ (- x)
2003
= x
2003
- x
2003
= 0.
Bài 4(14) : Cho ∆ABC. Trên các tia đối của các tia CB, AC, BA lần lượt lấy các điểm A
1

, B
1
, C
1

sao cho AB
1
= BC
1
= CA
1
. Chứng minh rằng nếu ∆A
1
B
1
C
1
đều thì ∆ABC cũng đều.
Lời giải :
Giả sử ∆A
1
B
1
C
1
đều. Không mất tính tổng quát ta có thể giả thiết :
Khi đó : CB
1
≥ AC
1

≥ BA
1
hay CA ≥ AB ≥ BC (do AB
1
= BC
1
= CA
1
), suy ra
Mặt khác, từ (*) suy ra : (do ∆A
1
B
1
C
1
đều). Nhưng
Từ (1), (2) suy ra ∆ABC đều (đpcm).
Bài 5(14) : Cho ba điểm A, B, C thẳng hàng, AB = BC. Một đường tròn (O) đi qua A, B. Các tiếp
tuyến với (O) kẻ từ A, C cắt nhau tại S. T là tiếp điểm của SC và (O). SB cắt (O) tại E (E khác B).
Chứng minh rằng : ET // AB.
Lời giải :
Vì SA, ST tiếp xúc với (O) nên ta có : ∠ STE = ∠ SBT ; ∠ SAE = ∠ SBA => ∆STE đồng dạng với
∆SBT ; ∆SAE đồng dạng với ∆ABA
Mặt khác, vì tứ giác AETB nội tiếp nên : ∠ TEA = ∠ TBC (2)
Từ (1), (2) ta có : ∆TEA đồng dạng với ∆TBC => ∠ EAT = ∠ BCT.
Từ đó, với chú ý rằng : ∠ EAT = ∠ ETS, ta có : ∠ BCT = ∠ ETS => ET // AB (hai góc đồng vị
bằng nhau).
Bài 4(15) : Cho tam giác ABC có AB > AC. Trên các cạnh AB, AC lấy các điểm N, M tương ứng,
sao cho AN = AM. Gọi O là giao điểm của BM, CN.
Chứng minh rằng : OB > OC.

Lời giải : (của bạn Dương Cao Nguyên)
Vì AB > AC > AM = AN nên tồn tại điểm K thuộc đoạn BN sao cho : AK = AC. Gọi L là giao
điểm của KM và CN. Vì K thuộc đoạn BN nên L thuộc đoạn ON => ∠ OMN > ∠LMN.
Mặt khác, dễ thấy tam giác LMN cân tại L => ∠ LMN = ∠ LNM.
Vậy : ∠ OMN > ∠ ONM => ON > OM (1).
∆AKM = ∆ACN (c.g.c) => KM = CN (2).
Vì AK = AC nên tam giác AKC cân tại A
=> ∠ AKC < 90
o
=> ∠ BKM > 90
o

=> ∠ BKM > ∠ KMB => BM > KM (3).
Từ (2) và (3) suy ra : BM > CN (4)
Từ (1) và (4) suy ra : BM - OM > CN - ON => OB > OC.

×